Let $a_1< cdots< a_nleq x$, where no $a_i$ divides product of others, show that $nleq pi(x)$.












1












$begingroup$


Let $a_1< a_2< cdots< a_nleq x$ be a set of positive integers such that no $a_i$ divides the product of the others. Prove that $nleq pi(x)$.




I have tried to argue by contradiction, which I assume $n> pi(x)$, then $a_1, a_2, cdots,a_n$ cannot be mutually relatively prime, so some of $a_i$ must divides product of other $a$'s, but I cannot give a very accurate explanation about it. Any suggestion?










share|cite|improve this question









$endgroup$

















    1












    $begingroup$


    Let $a_1< a_2< cdots< a_nleq x$ be a set of positive integers such that no $a_i$ divides the product of the others. Prove that $nleq pi(x)$.




    I have tried to argue by contradiction, which I assume $n> pi(x)$, then $a_1, a_2, cdots,a_n$ cannot be mutually relatively prime, so some of $a_i$ must divides product of other $a$'s, but I cannot give a very accurate explanation about it. Any suggestion?










    share|cite|improve this question









    $endgroup$















      1












      1








      1





      $begingroup$


      Let $a_1< a_2< cdots< a_nleq x$ be a set of positive integers such that no $a_i$ divides the product of the others. Prove that $nleq pi(x)$.




      I have tried to argue by contradiction, which I assume $n> pi(x)$, then $a_1, a_2, cdots,a_n$ cannot be mutually relatively prime, so some of $a_i$ must divides product of other $a$'s, but I cannot give a very accurate explanation about it. Any suggestion?










      share|cite|improve this question









      $endgroup$




      Let $a_1< a_2< cdots< a_nleq x$ be a set of positive integers such that no $a_i$ divides the product of the others. Prove that $nleq pi(x)$.




      I have tried to argue by contradiction, which I assume $n> pi(x)$, then $a_1, a_2, cdots,a_n$ cannot be mutually relatively prime, so some of $a_i$ must divides product of other $a$'s, but I cannot give a very accurate explanation about it. Any suggestion?







      prime-numbers analytic-number-theory






      share|cite|improve this question













      share|cite|improve this question











      share|cite|improve this question




      share|cite|improve this question










      asked Jan 25 at 11:23









      kelvin hong 方kelvin hong 方

      80018




      80018






















          2 Answers
          2






          active

          oldest

          votes


















          3












          $begingroup$

          Since no $a_i$ divides the product of the others, then for each fixed index $i$ there must be some prime $p_i$ such that $ineq jimplies v_{p_i}(a_i)>v_{p_i}(a_j)$. There may, of course, be several such primes...if so, let $p_i$ denote the least of them. Clearly $ineq jimplies p_ineq p_j$. In this way we have produced $n$ distinct primes, all less than $x$ so we are done.



          Note: here, as usual, for a prime $p$ and a natural number $a$ we define $v_p(a)$ to be the order to which $p$ divides $a$. Thus, for example, $v_3(18)=2$ and $v_5(18)=0$.






          share|cite|improve this answer











          $endgroup$













          • $begingroup$
            I have thought a while, but why $ineq j$ implies $p_ineq p_j$? Is the statement $ineq j$ implies $v_{p_i}(a_i)> v_{p_i}(a_j)$ true for all $jneq i$ or just some $jneq i$?
            $endgroup$
            – kelvin hong 方
            Jan 25 at 11:51






          • 1




            $begingroup$
            Suppose $p_i=p_j=p$. Then $v_p(a_i)>v_p(a_j)$ and $v_p(a_j)>v_p(a_i)$.
            $endgroup$
            – lulu
            Jan 25 at 11:52










          • $begingroup$
            thank, now I know how to do this.
            $endgroup$
            – kelvin hong 方
            Jan 25 at 11:55



















          1












          $begingroup$

          Assume to the contrary that there exists $x$ and $a_1<a_2<...<a_nleq x$ and no $a_i$ divides the product of the others and $n>pi(x)$. We pick such $a_1,a_2,...,a_n$ and $x$ such that the sum $a_1+a_2+...+a_n$ is minimum. Then we can show that $a_{i}$'s are pairwise coprime.(Suppose that $(a_i,a_j)=dneq 1$ Then replace (a_i,a_j) by (a_i/d,a_j/d) and consider the minimality of the sum $a_1+a_2+...+a_n$.) Then each $a_i$ has a prime divisor(as $a_ineq 1$) which does not divide any other $a_j$'s. Therefore each $a_i$ has a distinct prime divisor. Hence there exists at least $n$ primes that are less than or equal to $x$. This is a contradiction. Hence the conclusion follows.






          share|cite|improve this answer









          $endgroup$













          • $begingroup$
            Is it possible after we make the sum to be minimum, then some of $a_i$ becomes $1$?
            $endgroup$
            – kelvin hong 方
            Jan 25 at 11:48






          • 1




            $begingroup$
            In that case, 1 divides the product of the others.
            $endgroup$
            – NotoriousJuanG
            Jan 25 at 17:06











          Your Answer





          StackExchange.ifUsing("editor", function () {
          return StackExchange.using("mathjaxEditing", function () {
          StackExchange.MarkdownEditor.creationCallbacks.add(function (editor, postfix) {
          StackExchange.mathjaxEditing.prepareWmdForMathJax(editor, postfix, [["$", "$"], ["\\(","\\)"]]);
          });
          });
          }, "mathjax-editing");

          StackExchange.ready(function() {
          var channelOptions = {
          tags: "".split(" "),
          id: "69"
          };
          initTagRenderer("".split(" "), "".split(" "), channelOptions);

          StackExchange.using("externalEditor", function() {
          // Have to fire editor after snippets, if snippets enabled
          if (StackExchange.settings.snippets.snippetsEnabled) {
          StackExchange.using("snippets", function() {
          createEditor();
          });
          }
          else {
          createEditor();
          }
          });

          function createEditor() {
          StackExchange.prepareEditor({
          heartbeatType: 'answer',
          autoActivateHeartbeat: false,
          convertImagesToLinks: true,
          noModals: true,
          showLowRepImageUploadWarning: true,
          reputationToPostImages: 10,
          bindNavPrevention: true,
          postfix: "",
          imageUploader: {
          brandingHtml: "Powered by u003ca class="icon-imgur-white" href="https://imgur.com/"u003eu003c/au003e",
          contentPolicyHtml: "User contributions licensed under u003ca href="https://creativecommons.org/licenses/by-sa/3.0/"u003ecc by-sa 3.0 with attribution requiredu003c/au003e u003ca href="https://stackoverflow.com/legal/content-policy"u003e(content policy)u003c/au003e",
          allowUrls: true
          },
          noCode: true, onDemand: true,
          discardSelector: ".discard-answer"
          ,immediatelyShowMarkdownHelp:true
          });


          }
          });














          draft saved

          draft discarded


















          StackExchange.ready(
          function () {
          StackExchange.openid.initPostLogin('.new-post-login', 'https%3a%2f%2fmath.stackexchange.com%2fquestions%2f3086986%2flet-a-1-cdots-a-n-leq-x-where-no-a-i-divides-product-of-others-show-tha%23new-answer', 'question_page');
          }
          );

          Post as a guest















          Required, but never shown

























          2 Answers
          2






          active

          oldest

          votes








          2 Answers
          2






          active

          oldest

          votes









          active

          oldest

          votes






          active

          oldest

          votes









          3












          $begingroup$

          Since no $a_i$ divides the product of the others, then for each fixed index $i$ there must be some prime $p_i$ such that $ineq jimplies v_{p_i}(a_i)>v_{p_i}(a_j)$. There may, of course, be several such primes...if so, let $p_i$ denote the least of them. Clearly $ineq jimplies p_ineq p_j$. In this way we have produced $n$ distinct primes, all less than $x$ so we are done.



          Note: here, as usual, for a prime $p$ and a natural number $a$ we define $v_p(a)$ to be the order to which $p$ divides $a$. Thus, for example, $v_3(18)=2$ and $v_5(18)=0$.






          share|cite|improve this answer











          $endgroup$













          • $begingroup$
            I have thought a while, but why $ineq j$ implies $p_ineq p_j$? Is the statement $ineq j$ implies $v_{p_i}(a_i)> v_{p_i}(a_j)$ true for all $jneq i$ or just some $jneq i$?
            $endgroup$
            – kelvin hong 方
            Jan 25 at 11:51






          • 1




            $begingroup$
            Suppose $p_i=p_j=p$. Then $v_p(a_i)>v_p(a_j)$ and $v_p(a_j)>v_p(a_i)$.
            $endgroup$
            – lulu
            Jan 25 at 11:52










          • $begingroup$
            thank, now I know how to do this.
            $endgroup$
            – kelvin hong 方
            Jan 25 at 11:55
















          3












          $begingroup$

          Since no $a_i$ divides the product of the others, then for each fixed index $i$ there must be some prime $p_i$ such that $ineq jimplies v_{p_i}(a_i)>v_{p_i}(a_j)$. There may, of course, be several such primes...if so, let $p_i$ denote the least of them. Clearly $ineq jimplies p_ineq p_j$. In this way we have produced $n$ distinct primes, all less than $x$ so we are done.



          Note: here, as usual, for a prime $p$ and a natural number $a$ we define $v_p(a)$ to be the order to which $p$ divides $a$. Thus, for example, $v_3(18)=2$ and $v_5(18)=0$.






          share|cite|improve this answer











          $endgroup$













          • $begingroup$
            I have thought a while, but why $ineq j$ implies $p_ineq p_j$? Is the statement $ineq j$ implies $v_{p_i}(a_i)> v_{p_i}(a_j)$ true for all $jneq i$ or just some $jneq i$?
            $endgroup$
            – kelvin hong 方
            Jan 25 at 11:51






          • 1




            $begingroup$
            Suppose $p_i=p_j=p$. Then $v_p(a_i)>v_p(a_j)$ and $v_p(a_j)>v_p(a_i)$.
            $endgroup$
            – lulu
            Jan 25 at 11:52










          • $begingroup$
            thank, now I know how to do this.
            $endgroup$
            – kelvin hong 方
            Jan 25 at 11:55














          3












          3








          3





          $begingroup$

          Since no $a_i$ divides the product of the others, then for each fixed index $i$ there must be some prime $p_i$ such that $ineq jimplies v_{p_i}(a_i)>v_{p_i}(a_j)$. There may, of course, be several such primes...if so, let $p_i$ denote the least of them. Clearly $ineq jimplies p_ineq p_j$. In this way we have produced $n$ distinct primes, all less than $x$ so we are done.



          Note: here, as usual, for a prime $p$ and a natural number $a$ we define $v_p(a)$ to be the order to which $p$ divides $a$. Thus, for example, $v_3(18)=2$ and $v_5(18)=0$.






          share|cite|improve this answer











          $endgroup$



          Since no $a_i$ divides the product of the others, then for each fixed index $i$ there must be some prime $p_i$ such that $ineq jimplies v_{p_i}(a_i)>v_{p_i}(a_j)$. There may, of course, be several such primes...if so, let $p_i$ denote the least of them. Clearly $ineq jimplies p_ineq p_j$. In this way we have produced $n$ distinct primes, all less than $x$ so we are done.



          Note: here, as usual, for a prime $p$ and a natural number $a$ we define $v_p(a)$ to be the order to which $p$ divides $a$. Thus, for example, $v_3(18)=2$ and $v_5(18)=0$.







          share|cite|improve this answer














          share|cite|improve this answer



          share|cite|improve this answer








          edited Jan 25 at 13:24

























          answered Jan 25 at 11:29









          lulululu

          42.9k25080




          42.9k25080












          • $begingroup$
            I have thought a while, but why $ineq j$ implies $p_ineq p_j$? Is the statement $ineq j$ implies $v_{p_i}(a_i)> v_{p_i}(a_j)$ true for all $jneq i$ or just some $jneq i$?
            $endgroup$
            – kelvin hong 方
            Jan 25 at 11:51






          • 1




            $begingroup$
            Suppose $p_i=p_j=p$. Then $v_p(a_i)>v_p(a_j)$ and $v_p(a_j)>v_p(a_i)$.
            $endgroup$
            – lulu
            Jan 25 at 11:52










          • $begingroup$
            thank, now I know how to do this.
            $endgroup$
            – kelvin hong 方
            Jan 25 at 11:55


















          • $begingroup$
            I have thought a while, but why $ineq j$ implies $p_ineq p_j$? Is the statement $ineq j$ implies $v_{p_i}(a_i)> v_{p_i}(a_j)$ true for all $jneq i$ or just some $jneq i$?
            $endgroup$
            – kelvin hong 方
            Jan 25 at 11:51






          • 1




            $begingroup$
            Suppose $p_i=p_j=p$. Then $v_p(a_i)>v_p(a_j)$ and $v_p(a_j)>v_p(a_i)$.
            $endgroup$
            – lulu
            Jan 25 at 11:52










          • $begingroup$
            thank, now I know how to do this.
            $endgroup$
            – kelvin hong 方
            Jan 25 at 11:55
















          $begingroup$
          I have thought a while, but why $ineq j$ implies $p_ineq p_j$? Is the statement $ineq j$ implies $v_{p_i}(a_i)> v_{p_i}(a_j)$ true for all $jneq i$ or just some $jneq i$?
          $endgroup$
          – kelvin hong 方
          Jan 25 at 11:51




          $begingroup$
          I have thought a while, but why $ineq j$ implies $p_ineq p_j$? Is the statement $ineq j$ implies $v_{p_i}(a_i)> v_{p_i}(a_j)$ true for all $jneq i$ or just some $jneq i$?
          $endgroup$
          – kelvin hong 方
          Jan 25 at 11:51




          1




          1




          $begingroup$
          Suppose $p_i=p_j=p$. Then $v_p(a_i)>v_p(a_j)$ and $v_p(a_j)>v_p(a_i)$.
          $endgroup$
          – lulu
          Jan 25 at 11:52




          $begingroup$
          Suppose $p_i=p_j=p$. Then $v_p(a_i)>v_p(a_j)$ and $v_p(a_j)>v_p(a_i)$.
          $endgroup$
          – lulu
          Jan 25 at 11:52












          $begingroup$
          thank, now I know how to do this.
          $endgroup$
          – kelvin hong 方
          Jan 25 at 11:55




          $begingroup$
          thank, now I know how to do this.
          $endgroup$
          – kelvin hong 方
          Jan 25 at 11:55











          1












          $begingroup$

          Assume to the contrary that there exists $x$ and $a_1<a_2<...<a_nleq x$ and no $a_i$ divides the product of the others and $n>pi(x)$. We pick such $a_1,a_2,...,a_n$ and $x$ such that the sum $a_1+a_2+...+a_n$ is minimum. Then we can show that $a_{i}$'s are pairwise coprime.(Suppose that $(a_i,a_j)=dneq 1$ Then replace (a_i,a_j) by (a_i/d,a_j/d) and consider the minimality of the sum $a_1+a_2+...+a_n$.) Then each $a_i$ has a prime divisor(as $a_ineq 1$) which does not divide any other $a_j$'s. Therefore each $a_i$ has a distinct prime divisor. Hence there exists at least $n$ primes that are less than or equal to $x$. This is a contradiction. Hence the conclusion follows.






          share|cite|improve this answer









          $endgroup$













          • $begingroup$
            Is it possible after we make the sum to be minimum, then some of $a_i$ becomes $1$?
            $endgroup$
            – kelvin hong 方
            Jan 25 at 11:48






          • 1




            $begingroup$
            In that case, 1 divides the product of the others.
            $endgroup$
            – NotoriousJuanG
            Jan 25 at 17:06
















          1












          $begingroup$

          Assume to the contrary that there exists $x$ and $a_1<a_2<...<a_nleq x$ and no $a_i$ divides the product of the others and $n>pi(x)$. We pick such $a_1,a_2,...,a_n$ and $x$ such that the sum $a_1+a_2+...+a_n$ is minimum. Then we can show that $a_{i}$'s are pairwise coprime.(Suppose that $(a_i,a_j)=dneq 1$ Then replace (a_i,a_j) by (a_i/d,a_j/d) and consider the minimality of the sum $a_1+a_2+...+a_n$.) Then each $a_i$ has a prime divisor(as $a_ineq 1$) which does not divide any other $a_j$'s. Therefore each $a_i$ has a distinct prime divisor. Hence there exists at least $n$ primes that are less than or equal to $x$. This is a contradiction. Hence the conclusion follows.






          share|cite|improve this answer









          $endgroup$













          • $begingroup$
            Is it possible after we make the sum to be minimum, then some of $a_i$ becomes $1$?
            $endgroup$
            – kelvin hong 方
            Jan 25 at 11:48






          • 1




            $begingroup$
            In that case, 1 divides the product of the others.
            $endgroup$
            – NotoriousJuanG
            Jan 25 at 17:06














          1












          1








          1





          $begingroup$

          Assume to the contrary that there exists $x$ and $a_1<a_2<...<a_nleq x$ and no $a_i$ divides the product of the others and $n>pi(x)$. We pick such $a_1,a_2,...,a_n$ and $x$ such that the sum $a_1+a_2+...+a_n$ is minimum. Then we can show that $a_{i}$'s are pairwise coprime.(Suppose that $(a_i,a_j)=dneq 1$ Then replace (a_i,a_j) by (a_i/d,a_j/d) and consider the minimality of the sum $a_1+a_2+...+a_n$.) Then each $a_i$ has a prime divisor(as $a_ineq 1$) which does not divide any other $a_j$'s. Therefore each $a_i$ has a distinct prime divisor. Hence there exists at least $n$ primes that are less than or equal to $x$. This is a contradiction. Hence the conclusion follows.






          share|cite|improve this answer









          $endgroup$



          Assume to the contrary that there exists $x$ and $a_1<a_2<...<a_nleq x$ and no $a_i$ divides the product of the others and $n>pi(x)$. We pick such $a_1,a_2,...,a_n$ and $x$ such that the sum $a_1+a_2+...+a_n$ is minimum. Then we can show that $a_{i}$'s are pairwise coprime.(Suppose that $(a_i,a_j)=dneq 1$ Then replace (a_i,a_j) by (a_i/d,a_j/d) and consider the minimality of the sum $a_1+a_2+...+a_n$.) Then each $a_i$ has a prime divisor(as $a_ineq 1$) which does not divide any other $a_j$'s. Therefore each $a_i$ has a distinct prime divisor. Hence there exists at least $n$ primes that are less than or equal to $x$. This is a contradiction. Hence the conclusion follows.







          share|cite|improve this answer












          share|cite|improve this answer



          share|cite|improve this answer










          answered Jan 25 at 11:30









          NotoriousJuanGNotoriousJuanG

          843




          843












          • $begingroup$
            Is it possible after we make the sum to be minimum, then some of $a_i$ becomes $1$?
            $endgroup$
            – kelvin hong 方
            Jan 25 at 11:48






          • 1




            $begingroup$
            In that case, 1 divides the product of the others.
            $endgroup$
            – NotoriousJuanG
            Jan 25 at 17:06


















          • $begingroup$
            Is it possible after we make the sum to be minimum, then some of $a_i$ becomes $1$?
            $endgroup$
            – kelvin hong 方
            Jan 25 at 11:48






          • 1




            $begingroup$
            In that case, 1 divides the product of the others.
            $endgroup$
            – NotoriousJuanG
            Jan 25 at 17:06
















          $begingroup$
          Is it possible after we make the sum to be minimum, then some of $a_i$ becomes $1$?
          $endgroup$
          – kelvin hong 方
          Jan 25 at 11:48




          $begingroup$
          Is it possible after we make the sum to be minimum, then some of $a_i$ becomes $1$?
          $endgroup$
          – kelvin hong 方
          Jan 25 at 11:48




          1




          1




          $begingroup$
          In that case, 1 divides the product of the others.
          $endgroup$
          – NotoriousJuanG
          Jan 25 at 17:06




          $begingroup$
          In that case, 1 divides the product of the others.
          $endgroup$
          – NotoriousJuanG
          Jan 25 at 17:06


















          draft saved

          draft discarded




















































          Thanks for contributing an answer to Mathematics Stack Exchange!


          • Please be sure to answer the question. Provide details and share your research!

          But avoid



          • Asking for help, clarification, or responding to other answers.

          • Making statements based on opinion; back them up with references or personal experience.


          Use MathJax to format equations. MathJax reference.


          To learn more, see our tips on writing great answers.




          draft saved


          draft discarded














          StackExchange.ready(
          function () {
          StackExchange.openid.initPostLogin('.new-post-login', 'https%3a%2f%2fmath.stackexchange.com%2fquestions%2f3086986%2flet-a-1-cdots-a-n-leq-x-where-no-a-i-divides-product-of-others-show-tha%23new-answer', 'question_page');
          }
          );

          Post as a guest















          Required, but never shown





















































          Required, but never shown














          Required, but never shown












          Required, but never shown







          Required, but never shown

































          Required, but never shown














          Required, but never shown












          Required, but never shown







          Required, but never shown







          Popular posts from this blog

          Mario Kart Wii

          What does “Dominus providebit” mean?

          Antonio Litta Visconti Arese